Login

Welcome, Guest. Please login or register.

May 18, 2024, 09:47:13 am

Author Topic: HSC Physics Question Thread  (Read 1042740 times)  Share 

0 Members and 6 Guests are viewing this topic.

yattmoani

  • Adventurer
  • *
  • Posts: 21
  • Respect: 0
Re: Physics Question Thread
« Reply #2460 on: July 23, 2017, 06:36:26 pm »
+1
Thank you very much for your reply! Your explanation was perfect :)

bsdfjnlkasn

  • Forum Obsessive
  • ***
  • Posts: 417
  • Respect: +28
Re: Physics Question Thread
« Reply #2461 on: July 25, 2017, 09:02:45 am »
0
Hey there!

Was hoping to get some help with the following question :)


EDIT: Could I also get a thorough answer to this following 4 marker? I'm just not sure how Lenz's law is relevant here and the order of all the changes is a bit confusing...

Here is what I understand:

Because current now flows through the solenoid, it will produce a north pole at the bottom (RHCurlR). Then the ring also experiences a changing magnetic flux because there has been a south pole produced at the top. So in order for the ring to oppose this change, it will need to produce a north pole acting down to weaken/cancel the new magnetic field (Lenz's law) around the ring. Then using the curl rule again, the induced current will be clockwise (thumb down, fingers around). BUT I'm not sure what happens next to give me an upwards force. The provided answer is super vague so would love some clarity.

2. How would you solve the projectile part of this question? I need to find the time it will take for the stone to hit the water.


3. I'm not sure why for part b) you can't use vy2 = uy2 + 2aydeltaY?


4. Another projectile  :)

I'm quite confused about what working out is happening after the initial velocity is calculated, because how does finding the angle to be 24° verify the table which states 22°? I'm also not really sure why they've used v = u+at

5. Lol really not getting these... Not sure what's happened after they found V (and also why it's negative)?

Any help would be super appreciated :)
« Last Edit: July 25, 2017, 11:18:54 am by bsdfjnlkasn »

beau77bro

  • Forum Obsessive
  • ***
  • Posts: 342
  • Respect: +6
Re: Physics Question Thread
« Reply #2462 on: July 27, 2017, 12:22:41 pm »
0
help please. I don't get these questions - explanation much appreciated

for Q5, does take off mean it hasn't left the ground yet? I assumed the definition of G-force was always just an expression of appraent weight as a multiple of true weight. but the answer is D???

for 7 it says B and i jsut don't get that, because the experiment couldn't read the effects of the aether - it didn't disprove but it couldn't get tangible results.

8. Is B, but i dont get why it's not D? is it just because it's cutting more? isn't it awlays to do with change in flux not amount of?

12. I just don;t get this. doesn't back emf get produced until it completely negates the input - thus preventing acceleration. Can someone explain how Back Emf relates to current and voltage in a motor when it's running i feel like my understanding is very holey around there. like why is there still current when no Emf?

15. if the force is increasing but at a lesser rate, doesn't that mean the charge must be increasing. The Answer is D like dahel.
« Last Edit: July 27, 2017, 12:53:00 pm by beau77bro »

beau77bro

  • Forum Obsessive
  • ***
  • Posts: 342
  • Respect: +6
Re: Physics Question Thread
« Reply #2463 on: July 27, 2017, 12:23:26 pm »
0
and this as i couldnt post all three.

jamonwindeyer

  • Honorary Moderator
  • Great Wonder of ATAR Notes
  • *******
  • Posts: 10150
  • The lurker from the north.
  • Respect: +3108
Re: Physics Question Thread
« Reply #2464 on: July 27, 2017, 12:34:56 pm »
0
Hey there!
Was hoping to get some help with the following question :)

Hey! This requires the formula:



This formula links magnetic flux to magnetic field strength (B) and area (A). Given the initial values of B and A, the initial flux is:



If we halve the area, we halve the flux, so the change in flux is roughly equal to 0.125 Webers. The induced EMF is therefore given by Faraday's Law:



Quote
EDIT: Could I also get a thorough answer to this following 4 marker? I'm just not sure how Lenz's law is relevant here and the order of all the changes is a bit confusing...

Here is what I understand:

Because current now flows through the solenoid, it will produce a north pole at the bottom (RHCurlR). Then the ring also experiences a changing magnetic flux because there has been a south pole produced at the top. So in order for the ring to oppose this change, it will need to produce a north pole acting down to weaken/cancel the new magnetic field (Lenz's law) around the ring. Then using the curl rule again, the induced current will be clockwise (thumb down, fingers around). BUT I'm not sure what happens next to give me an upwards force. The provided answer is super vague so would love some clarity.

You've got almost all of it - The induced current in the ring produces another magnetic field as you say. This interacts with the field produced by the coil and the ring is repelled ;D

====

Sorry, I'm a little confused as to how the rest of your screenshots line up with your questions (and there might be some missing?), you might need to clarify for me :)

jakesilove

  • HSC Lecturer
  • Honorary Moderator
  • Part of the furniture
  • *******
  • Posts: 1941
  • "Synergising your ATAR potential"
  • Respect: +196
Re: Physics Question Thread
« Reply #2465 on: July 27, 2017, 12:36:24 pm »
0
Hey there!

Was hoping to get some help with the following question :)


EDIT: Could I also get a thorough answer to this following 4 marker? I'm just not sure how Lenz's law is relevant here and the order of all the changes is a bit confusing...

Here is what I understand:

Because current now flows through the solenoid, it will produce a north pole at the bottom (RHCurlR). Then the ring also experiences a changing magnetic flux because there has been a south pole produced at the top. So in order for the ring to oppose this change, it will need to produce a north pole acting down to weaken/cancel the new magnetic field (Lenz's law) around the ring. Then using the curl rule again, the induced current will be clockwise (thumb down, fingers around). BUT I'm not sure what happens next to give me an upwards force. The provided answer is super vague so would love some clarity.

2. How would you solve the projectile part of this question? I need to find the time it will take for the stone to hit the water.


3. I'm not sure why for part b) you can't use vy2 = uy2 + 2aydeltaY?


4. Another projectile  :)

I'm quite confused about what working out is happening after the initial velocity is calculated, because how does finding the angle to be 24° verify the table which states 22°? I'm also not really sure why they've used v = u+at

5. Lol really not getting these... Not sure what's happened after they found V (and also why it's negative)?

Any help would be super appreciated :)

Maybe I'm looking at it wrong, but your questions don't seem to correspond to attachments. Please make this clearer, so we can better get to your questions. This could just be me failing to understand your explanation, but yeah.
ATAR: 99.80

Mathematics Extension 2: 93
Physics: 93
Chemistry: 93
Modern History: 94
English Advanced: 95
Mathematics: 96
Mathematics Extension 1: 98

Studying a combined Advanced Science/Law degree at UNSW

jamonwindeyer

  • Honorary Moderator
  • Great Wonder of ATAR Notes
  • *******
  • Posts: 10150
  • The lurker from the north.
  • Respect: +3108
Re: Physics Question Thread
« Reply #2466 on: July 27, 2017, 12:47:04 pm »
0
help please. I don't get these questions - explanation much appreciated

Hey!

Question 5: This is really just about knowing the definition. C is correct - G-force is about expressing apparent weight force under a certain acceleration in terms of weight force on earth :)

Question 7: Again, really just about knowing this fact. The Michelson-Morley experiment produced a null result, meaning it couldn't detect what it was looking for (which was the Aether Wind) :)

Question 8: Recall that the induced emf is proportional to the rate of change of magnetic flux, that is:



Now magnetic flux can be related to magnetic field strength through \(\phi=BA\). This should immediately suggest that a bigger coil rotating through a stronger magnetic field will give a larger flux, meaning more induced emf. More turns in the coil will have a similar effect, and spinning it with a greater frequency will make the change faster (increasing the rate of change). So, we want EVERYTHING to get bigger - I'd go for B :)

Question 12: I think the answer to this question depends a bit on if we assume the motor is ideal (frictionless). But given the answers, I think there is friction. In which case, it is fairly simple (I think, almost too easy?) - We need an AC voltage to drive an AC motor. So, B!

and this as i couldnt post all three.

The law for force experienced by a charge in an electric field:



Notice the similarity to a regular straight line, where \(y=F\) and \(E=x\). However, the line on the graph isn't straight. So clearly, the force isn't proportional to electric field - We can see that the line starts to taper off for larger fields - The force isn't changing much even though the field is. This means that the particle must be losing charge - The answer is D :)

beau77bro

  • Forum Obsessive
  • ***
  • Posts: 342
  • Respect: +6
Re: Physics Question Thread
« Reply #2467 on: July 27, 2017, 12:58:22 pm »
0
Hey!

Question 5: This is really just about knowing the definition. C is correct - G-force is about expressing apparent weight force under a certain acceleration in terms of weight force on earth :)

Question 7: Again, really just about knowing this fact. The Michelson-Morley experiment produced a null result, meaning it couldn't detect what it was looking for (which was the Aether Wind) :)

Question 8: Recall that the induced emf is proportional to the rate of change of magnetic flux, that is:



Now magnetic flux can be related to magnetic field strength through \(\phi=BA\). This should immediately suggest that a bigger coil rotating through a stronger magnetic field will give a larger flux, meaning more induced emf. More turns in the coil will have a similar effect, and spinning it with a greater frequency will make the change faster (increasing the rate of change). So, we want EVERYTHING to get bigger - I'd go for B :)

Question 12: I think the answer to this question depends a bit on if we assume the motor is ideal (frictionless). But given the answers, I think there is friction. In which case, it is fairly simple (I think, almost too easy?) - We need an AC voltage to drive an AC motor. So, B!

The law for force experienced by a charge in an electric field:



Notice the similarity to a regular straight line, where \(y=F\) and \(E=x\). However, the line on the graph isn't straight. So clearly, the force isn't proportional to electric field - We can see that the line starts to taper off for larger fields - The force isn't changing much even though the field is. This means that the particle must be losing charge - The answer is D :)

5 was D, 7 was B, 8 i get now, so it's not that it's changing at a greater rate - that change is bigger meaning more EMF.

12 i still don't get because what about back emf?

15. OMG I SEE - YOU TREAT IT LIKE A LINE. SO q much be decreasing - should've written it as the relationship. thankyou

But yea 5 and 7 have me stumped and i don't quite get 12.

also i modified the last one - but don't worry i ask the same questions here.
Thanks jamon!!!!

jamonwindeyer

  • Honorary Moderator
  • Great Wonder of ATAR Notes
  • *******
  • Posts: 10150
  • The lurker from the north.
  • Respect: +3108
Re: Physics Question Thread
« Reply #2468 on: July 27, 2017, 01:05:10 pm »
0
5 was D, 7 was B

No way either of those are correct - In honesty Q5 I was a little unsure because it is very much a technicality, but it can't be D, and no way 7 is B === 7 is definitely D :)

Quote
12 i still don't get because what about back emf?

At maximum speed there is definitely back EMF that opposes the supply, but maximum implies the back emf (plus friction) has balanced with the supply. The waveform looks the same but it has a smaller magnitude - That's my interpretation at least. What would you think Back EMF would do?

beau77bro

  • Forum Obsessive
  • ***
  • Posts: 342
  • Respect: +6
Re: Physics Question Thread
« Reply #2469 on: July 27, 2017, 01:28:37 pm »
0
No way either of those are correct - In honesty Q5 I was a little unsure because it is very much a technicality, but it can't be D, and no way 7 is B === 7 is definitely D :)

At maximum speed there is definitely back EMF that opposes the supply, but maximum implies the back emf (plus friction) has balanced with the supply. The waveform looks the same but it has a smaller magnitude - That's my interpretation at least. What would you think Back EMF would do?

OK THANKGOD I WAS QUESTIONING MY EXISTENCE.

anyway I always thought Back emf just negated input, making it 0 - i never thought or i dont remember about frictions part? i see now Friction slows the rotor - reducing the back EMF induced? and thus leaving the input voltage supplying a lessened voltage. woudl that be correct - i just never thought of friction playing a part without reference.

thanks again

jamonwindeyer

  • Honorary Moderator
  • Great Wonder of ATAR Notes
  • *******
  • Posts: 10150
  • The lurker from the north.
  • Respect: +3108
Re: Physics Question Thread
« Reply #2470 on: July 27, 2017, 01:29:52 pm »
0
OK THANKGOD I WAS QUESTIONING MY EXISTENCE.

anyway I always thought Back emf just negated input, making it 0 - i never thought or i dont remember about frictions part? i see now Friction slows the rotor - reducing the back EMF induced? and thus leaving the input voltage supplying a lessened voltage. woudl that be correct - i just never thought of friction playing a part without reference.

thanks again

Yep, so without friction the back emf would cancel the input and give us zero at max speed. But there is no blank graph option, so we assume friction plays a role, which means we must continually supply at least a small AC voltage to overcome that friction (even after back emf) :)

beau77bro

  • Forum Obsessive
  • ***
  • Posts: 342
  • Respect: +6
Re: Physics Question Thread
« Reply #2471 on: July 28, 2017, 08:27:50 am »
0
help pls. I dont understand

Mymy409

  • Trailblazer
  • *
  • Posts: 45
  • "Stars can't shine without darkness."
  • Respect: 0
Re: Physics Question Thread
« Reply #2472 on: July 28, 2017, 06:59:42 pm »
0
Do we have to know about tension and other prelim concepts in HSC Physics?

beau77bro

  • Forum Obsessive
  • ***
  • Posts: 342
  • Respect: +6
Re: Physics Question Thread
« Reply #2473 on: July 29, 2017, 09:00:37 am »
0
Do we have to know about tension and other prelim concepts in HSC Physics?

they sometimes pop up in trials, like extra content to check your knowledge - usually it's pretty minor understanding though, and I haven't much if any in hscs but Im sure Jake and Jamon have a better idea

beau77bro

  • Forum Obsessive
  • ***
  • Posts: 342
  • Respect: +6
Re: Physics Question Thread
« Reply #2474 on: July 29, 2017, 09:02:25 am »
0
more help needed. sorry for being a hassle, but the answers say the wavelength should half for this, even though the speed is kept constant. I disagree, but Im not sure- the amplitude should double but yea